50,400 different anagrams can be made from the letters in the word statistics.
Given word : STATISTICS
Anagrams are words formed by jumbling the positions of the letters given in the word.
For such problems we do factorial of number of words and divide by the repeated letters factorials.
Total number of letters = 10
They can be permutated among themselves in 10! ways.
Some letters are repeated in word STATISTICS
Since, there are 3 S's and 3 T's and 2 I's
So, Number of anagrams = [tex]\frac{10!}{3!.3!.2!}[/tex]
= [tex]\frac{10 \times 9 \times 8 \times 7 \times 6 \times 5 \times 4 \times 3!}{3! \times 3 \times 2 \times 1 \times 2 \times 1}[/tex]
= [tex]\frac{10 \times 9 \times 8 \times 7 \times 6 \times 5 \times 4}{3 \times 2 \times 2}[/tex]
= 10 × 9 × 8 × 7 × 2 × 5
= 50,400
50,400 different anagrams can be made from the letters in the word statistics.
Find out more information about anagrams here
brainly.com/question/14293907
#SPJ4
Consider a single spin of the spinner. A spinner contains 4 equal sections: 1, 2, 4 and 3. Sections 1 and 4 are shaded. The spinner is pointed at number 2. Which events are mutually exclusive
Answer:
Events that mutually exclusive are 1. Landing on a shaded portion and landing on a 3.2. Landing on an unshaded portion and landing on a number less than 2.
pls help will mark this brainlest
The ordered pairs (0,13) and (10, 0) are joined to best draw the line of best fit for the given scatter plot. So, option 4 is correct.
How to draw the line of best fit for a scatter plot?For the given scatter plot, to draw a line of best fit, the slope is to be calculated. The slope of the required line is calculated by
m = [n(∑xy) - (∑x)(∑y)]/[n(∑x²) - (∑x)²]
Where,
∑xy = sum of the product of x and y values
∑x = sum of x values
∑y = sum of y values
∑x² = sum of square values of x
n = total number of scatter points
And the y-intercept is calculated by
b = [∑y - m(∑x)]/n
Where m is the slope obtained above
Calculation:The given scatter plot has the coordinate points:
(0,14), (1, 11), (2, 9), (3, 10),(4, 7), (5, 7), (6, 5), (7, 5), (8, 3), (9, 1), (10, 0)
Such that n = 11
Then the required components are calculated as follows:
∑x = 0 + 1 + 2 + 3 + 4 + 5 + 6 + 7 + 8 + 9 + 10 = 55
∑y = 14 + 11 + 9 + 10 + 7 + 7 + 5 + 5 + 3 + 1 + 0 = 72
∑xy = (0 × 14) + (1 × 11) + (2 × 9) + (3 × 10) + (4 × 7) + (5 × 7) + (6 × 5) + (7 × 5) + (8 × 3) + (9 × 1) + (10 × 0) = 220
∑x² = 0² + 1² + 2² + 3² + 4² + 5² + 6² + 7² + 8² + 9² + 10² = 385
Then the slope is calculated as follows:
slope m = [n(∑xy) - (∑x)(∑y)]/[n(∑x²) - (∑x)²]
On substituting,
m = [11(220) - (55)(72)]/[11(385) - (55)²]
⇒ m = -14/11 = -1.2727273 ≅ -1.3
∴ m = -1.3
Then calculating the y-intercept:
we have b = [∑y - m(∑x)]/n
On substituting,
b = [72 - -1.3(55)]/11
∴ b = 13
Then the slope-intercept form of the required line is
y = -1.3x + 13
When x = 0,
y = -1.3(0) + 13 = 13
When y = 0,
0 = -1.3x + 13
⇒ 1.3x = 13
⇒ x = 13/1.3 = 10
Therefore, the coordinates (0, 13) and (10, 0) give the best draw for the line of best fit.
Learn more about the scatterplot and the line of best fit here:
https://brainly.com/question/1518824
#SPJ1
A company currently pays a dividend of $2.6 per share (d0 = $2.6). it is estimated that the company's dividend will grow at a rate of 24% per year for the next 2 years, and then at a constant rate of 8% thereafter. the company's stock has a beta of 1.8, the risk-free rate is 7.5%, and the market risk premium is 4.5%. what is your estimate of the stock's current price?
The current value of the stock is $53.413455.
What is the CAMP model?The CAMP model describes the relationship between systemic risk, or the general dangers of investing, and projected return on assets, specifically stocks.To find the current price of stock:
Stock’s current price: 53.41 dollars
First, we determine the stock's value using the CAPM model.
Ke = rf + (rm - rf)risk free, rf = 0.085.premium market = (rm - rf) = 0.045(nondiversifiable risk) = 1.3Then, Ke = 0.085 + 1.3(0.045).
Ke = 0.14350The current value of the future dividends must now be known:
D0 = 2.8D1 = D0 x (1 + g) = 2.8 * 1.23 = 3.444D2 = 3.444 x 1.23 = 4.2361200The next dividends, which are at perpetuity will be solved using the dividend growth model:
dividends/return-growth = Intrinsic valueIn this case, dividends will be:
4.23612 x 1.07 = 4.5326484Return will be calculated using the CAPM and g = 7%.
plug this into the Dividend grow model,
4.53264840.1435-0.07 = Intrinsic ValueValue of the dividends at perpetuity: 61.6686857Finally, it's crucial to remember that these statistics were calculated for the present year.
We must bring them to the present day using the present value of a lump sum:
principal/(1 + rate)time = PV3.444/(1 + 0.1435)1 = PV3.011805859 = PV4.23612/(1 + 0.1435)2 = PV3.239633762 = PV61.6686857/(1 + 0.1435)2 = PV47.16201531 = PVWe add them and get the value of the stock, 53.413455
Therefore, the current value of the stock is $53.413455.
Know more about the stock's current value here:
https://brainly.com/question/20813161
#SPJ4
The data set below has a lower quartile of 13 and an upper quartile of 37.
1, 12, 13, 15, 18, 20, 35, 37, 40, 78
Which statement is true about any outliers of the data set?
The correct option regarding the outliers of the data-set is given by:
The greatest value, 78, is the only outlier.
How to use the quartiles of a data-set to identitfy outliers?The median of the data-set separates the bottom half from the upper half, that is, it is the 50th percentile.The first quartile is the median of the first half of the data-set.The third quartile is the median of the second half of the data-set.The interquartile range is the difference of the third quartile with the first quartile.Measures that are more than 1.5 IQR from Q1 and Q3 are considered outliers.The IQR for this problem is:
IQR = 37 - 13 = 24.
Hence the bounds for outliers are:
Less than 13 - 1.5 x 24 = -23.Greater than 37 + 1.5 x 24 = 73,The options are:
No outliers.Only 1 is an outlier.Only 78 is an outlier.Both 1 and 78 are outliers.Hence the correct option is that only 78 is an outlier.
More can be learned about outliers of a data-set at https://brainly.com/question/17083142
#SPJ1
A scale measured a 4.5-pound brick as weighing 5.3 pounds. which measurement is more accurate but less precise than 5.3 pounds? 4.98 pounds 5 pounds 5.52 pounds 6 pounds
The correct option is B.
5 pound
The number (5.3) after Decimal is <5 so it round off to 5.
What is the property of rounding off numbers to one decimal place?It is the same to round a number to one decimal place as to the nearest tenths. At this instance, it is known which numeral is in the hundredths position. When the number in the hundredths place is higher than or equal to 5, the tenths digit is raised by one unit.
How do you round off decimals examples?A typical rule of thumb is to glance at the digit immediately to the right of the place value you want to round to and make your choice. For instance, rounding 5.1837 to the closest hundredth would result in 5.18 (because 3<5), whereas rounding 5.184 to the nearest thousandth would result in 5.184 (because 7>5).
To know more about the decimal number visit:
https://brainly.com/question/4708407
#SPJ4
I understand that the question your are looking for is:
A scale measured a 4.5-pound brick as weighing 5.3 pounds. which measurement is more accurate but less precise than 5.3 pounds?
A. 4.98 pounds
B. 5 pounds
C. 5.52 pounds
D. 6 pounds
Evaluate the following series:
This is a telescoping sum. The K-th partial sum is
[tex]S_K = \displaystyle \sum_{k=1}^K \left(\frac1{\sqrt{k+1}} - \frac1{\sqrt{k+3}}\right) \\\\ ~~~= \left(\frac1{\sqrt2} - \frac1{\sqrt4}\right) + \left(\frac1{\sqrt3} - \frac1{\sqrt5}\right) + \left(\frac1{\sqrt4} - \frac1{\sqrt6}\right) + \left(\frac1{\sqrt5} - \frac1{\sqrt7}\right) + \cdots \\\\ ~~~~~~~~+ \left(\frac1{\sqrt{K-1}} - \frac1{\sqrt{K+1}}\right) \\\\ ~~~~~~~~+ \left(\frac1{\sqrt K} - \frac1{\sqrt{K+2}}\right) + \left(\frac1{\sqrt{K+1}} - \frac1{\sqrt{K+3}}\right)[/tex]
[tex]\displaystyle = \frac1{\sqrt2} + \frac1{\sqrt3} - \frac1{\sqrt{K+2}} - \frac1{\sqrt{K+3}}[/tex]
As [tex]K\to\infty[/tex], the two trailing terms will converge to 0, and the overall infinite sum will converge to
[tex]\displaystyle \sum_{k=1}^\infty \left(\frac1{\sqrt{k+1}} - \frac1{\sqrt{k+3}}\right) = \lim_{k\to\infty} S_k = \boxed{\frac1{\sqrt2} + \frac1{\sqrt3}}[/tex]
By the limit comparison test, the expression √[1 / (1 + 1 / k)] - √[1 / (1 + 3 / k)] has a limit, then the expression [1 / √(k + 1)] / [1 /√k] - [1 / √(k + 3)] / [1 /√k] has a limit and the series ∑ [1 / √(k + 1)] - ∑ [1 / √(k + 3)] is convergent.
Is the series convergent?
Herein we have a series that involves radical components. First, we simplify the expression given:
∑ [1 / √(k + 1) - 1 / √(k + 3)] = ∑ [1 / √(k + 1)] - ∑ [1 / √(k + 3)]
The convergence of the series can be proved by the limit comparison test, where each component of the subtraction of the series is compared with a series that is convergent. We notice that both 1 / √(k + 1) and 1 / √(k + 3) resembles the expresion 1 /√k. Then, we have the following subtraction of ratios:
[1 / √(k + 1)] / [1 /√k] - [1 / √(k + 3)] / [1 /√k]
√k / √(k + 1) - √k / √(k + 3)
√[k / (k + 1)] - √[k / (k + 3)]
Then, by using the limit property for rational functions we find the following result for n → + ∞:
√[1 / (1 + 0)] - √[1 / (1 + 0)]
√1 - √1
1 - 1
0
By the limit comparison test, the expression √[1 / (1 + 1 / k)] - √[1 / (1 + 3 / k)] has a limit, then the expression [1 / √(k + 1)] / [1 /√k] - [1 / √(k + 3)] / [1 /√k] has a limit and the series ∑ [1 / √(k + 1)] - ∑ [1 / √(k + 3)] is convergent.
Remark
The statement is incomplete and complete form cannot be found, therefore, we decided to determine if the series is convergent or not.
To learn more on convergence: https://brainly.com/question/15415793
#SPJ1
Help on this math problem!! if there is any work that can be shown it would be great thanks!!!
How do I solve this question.
Hello, the solution of the geometry problem is in this photo.
AOC = 136°
BOC = 44°
(MID SCHOOL ALGEBRA)
can somebody explain to me from this image, why the -13x and +6 becomes +13x and -6?
Answer:
- (2x^2 - 13x + 6)
The above expression is in brackets and a negative sign is outside of the brackets. This means that when expanding the expression, we have to multiply each number with the negative sign (each number would switch their signs to the opposite one). Therefore, 2x^2 becomes -2x^2, -13x becomes 13x and 6 becomes -6.
Which of the following is a solution to the equation
3/4
x-12 = -18?
1) 4.5
2) -8
3) -22.5
4) -40
Answer: B) -8
Step-by-step explanation:
We add 12 both sides
3/4 x = -18 + 12Add −18 and 12 to get −6.
3/4 x = -6Multiply both sides by 3/4 by the reciprocal of 3/4.
x = -6 × (3/4)Express a -6 x (3/4) as a single fraction.
x = -6 × 4 / 3Multiply −6 and 4 to get −24.
x = -24 / 3Divide −24 by 3 to get −8.
x = -8Answer: Option "2" is correct. ✅
Type the correct answer in each box. Round your answers to two decimal places.
Subtract vector v = <2, -3> from vector u = <5, 2>.
The magnitude of the resulting vector, u - v, is approximately __
and its angle of direction is approximately ___
The magnitude of the resulting vector, u - v, is approximately 5.83
and its angle of direction is approximately 59.04°.
How to find the magnitude of the resulting vector?We want to subtract vector v from vector u.
We are given;
v = <2, -3> = 2i - 3j
u = <5, 2> = 5i + 2j
u - v = 5i + 2j - (2i - 3j)
= 5i + 2j - 2i + 3j
= 3i + 5j
Resultant vector = √(3² + 5²)
Resultant vector = √34 ≈ 5.83
Angle of direction of resultant vector is;
tan θ = (5/3)
θ = tan⁻¹(5/3)
θ = 59.04°
Read more about Magnitude of Vector at; https://brainly.com/question/3184914
#SPJ1
What number could be added to 0.40 ml for the level of precision to be 0.01 ml? check all that apply 0.2 ml 0.154 ml 6 ml 2.02 ml 8.8331 ml
The following volumes of fluid can be added to 0.40 mL of liquid for the level of precision of 0.01 mL: a) 0.2 mL, b) 6 mL, c) 2.02 mL. The amounts 0.154 mL and 8.8331 mL are not possible due to given level of precision.
What is the amount of liquid to be added to sample according to a given precision?
If the measuring has a level of precision of 0.01 ml, this means that the measured quantities are only sensible to the smallest hundreths. Any change less than 0.01 ml and any decimal less than a hundreths are "invisible" for measuring processes.
Hence, the following volumes of fluid can be added to 0.40 mL of liquid for the level of precision of 0.01 mL: a) 0.2 mL, b) 6 mL, c) 2.02 mL. The amounts 0.154 mL and 8.8331 mL are not possible due to given level of precision.
To learn more on precision: https://brainly.com/question/1311561
#SPJ1
Answer:
B,D,E
Step-by-step explanation:
which number set(s) does -10 belong to
irrational numbers
whole numbers
rational numbers
integers
real numbers
counting or natural numbers
No number set describes this number.
The number set(s) that - 10 belong to are rational numbers, integers and real numbers. Option C, D and E
Number sets of negative numbersA rational number can be defined as a number expressed as the ratio of two integers, where the denominator is not be equal to zer0
- 10 can be written as = 1/ 10
Integers are whole number that could be positive, negative and even zero
- 10 is a negative whole number
Real numbers are numbers with continuous quantity that can represent distance along a number line
-10 can represent distance along a number line.
Thus, the number set(s) that - 10 belong to are rational numbers, integers and real numbers. Option C, D and E
Learn more about real numbers here:
https://brainly.com/question/2515651
#SPJ1
I NEED HELP WITH MY MATH PLEASE
Which equation represents y = −x2 + 4x − 1 in vertex form?
The vertex form of the equation y = -x^2 + 4x - 1 is y = -(x - 2)^2 + 3
How to determine the vertex form of the quadratic equation?The quadratic equation is given as:
y = -x^2 + 4x - 1
Differentiate the above quadratic equation.
This is done with respect to x by first derivative
So, we have:
y' = -2x + 4
Set the derivative to 0
-2x + 4 = 0
Subtract 4 from both sides of the equation
-2x + 4 - 4 = 0 - 4
Evaluate the difference in the above equation
-2x = -4
Divide both sides of the above equation by -2
x = 2
Rewrite as
h = 2
Substitute 2 for x in the equation y = -x^2 + 4x - 1
y = -2^2 + 4 *2 - 1
Evaluate the equation
y = 3
Rewrite as:
k = 3
A quadratic equation in vertex form is represented as:
y = a(x - h)^2 + k
So, we have:
y = a(x - 2)^2 + 3
In the equation y = -x^2 + 4x - 1, a = -1
So, we have:
y = -(x - 2)^2 + 3
Hence, the vertex form of the equation y = -x^2 + 4x - 1 is y = -(x - 2)^2 + 3
Read more about vertex form at
https://brainly.com/question/18797214
#SPJ1
The quotient equals the divisor, then the dividend equals the
(A) √divisor
(B) divisor
(C) divisor²
(D) quotient
Look at the picture of a scaffold used to support construction workers. The height of the scaffold can be changed by adjusting two slanting rods, one of which, labeled PR, is shown:
A support structure is shown in which a right triangle PQR is formed with the right angle at Q. The length of PQ is shown as 14 feet, and the length of QR is shown as 6 feet..
Part A: What is the approximate length of rod PR? Round your answer to the nearest hundredth. Explain how you found your answer, stating the theorem you used. Show all your work. (5 points)
Part B: The length of rod PR is adjusted to 16 feet. If width PQ remains the same, what is the approximate new height QR of the scaffold? Round your answer to the nearest hundredth. Show all your work. (5 points)
Part A
Using the Pythagorean on the right triangle PQR, with PQ and QR as the legs and PR as the hypotenuse,
[tex]14^2 +6^2 =(PR)^2\\\\(PR)=\sqrt{14^2 +6^2}\\\\PR \approx \boxed{15.23 \text{ ft}}[/tex]
Part B
[tex](QR)^2 +6^2 =16^2\\\\(QR)^2 =16^2 -6^2\\\\QR=\sqrt{16^2 -6^2}\\\\QR \approx \boxed{14.83 \text{ ft}}[/tex]
The table below shows the birth rate (per 1000) per year in the United States according to data from the National Center for Health Statistics. Let x represent the number of years since 2000 with x = 0 representing the year 2000. Let y represent the birth rate per 1000 population. Write the slope-intercept form of the equation for the line of fit using the points representing 2001 and 2010. Round to the nearest hundredth.
The regression linear function, in slope-intercept form, is given by:
y = -0.07x + 14.5
How to find the equation of linear regression using a calculator?To find the equation, we need to insert the points (x,y) in the calculator.
For this problem, the points are given as follows:
(0, 14.7), (1, 14.5), (2, 14.4), (3, 14.1), (4, 14), (5, 14.1), (6, 14.1), (7, 14.2), (8, 14), (9, 13.8), (10, 14).
Inserting these points into a calculator, the regression linear function, in slope-intercept form, is given by:
y = -0.07x + 14.5
More can be learned about a linear regression equation at https://brainly.com/question/16793283
#SPJ1
Answer:
y = -0.07x + 14.5
Step-by-step explanation:
if 7 cosec^2 theta-9 cot^2theta=7 then what is the value of tantheta
[tex]\displaystyle\\Answer:\theta=\frac{\pi }{2}+\pi n.[/tex]
Step-by-step explanation:
[tex]\displaystyle\\7*cosec^2\theta-9*cot^2\theta=7\\7-7*cosec^2\theta+9*cot^2\theta=0\\7-\frac{7}{sin^2\theta}+9*\frac{cos^2\theta}{sin^2\theta} } =0\\\frac{7*sin^2\theta-7+9*cos^2\theta}{sin^2\theta} =0\\\frac{-7*(1-sin^2\theta)+9*cos^2\theta}{sin^2\theta} =0\\\frac{-7*cos^2\theta+9*cos^2\theta}{sin^2\theta} =0\\\frac{2*cos^2\theta}{sin^2\theta}=0\\[/tex]
[tex]2*cot^2\theta=0\\Divide\ the\ right\ and\ initial\ parts\ by\ 2:\\cot^2\theta=0\\cot\theta=0\\\theta=\frac{\pi }{2}+\pi n\ \ \ (n=0,\ 1,\ 2,\ 3\ ...).[/tex]
Alex travels 46
miles per hour for
3.2 hours. How far
has he gone?
Answer: 147.2 miles
Step-by-step explanation:
We can answer this by knowing the formula [tex]speed=\frac{distance}{time}[/tex]. Here, the speed is 46 miles/hour and the time is 3.2 hours. Let's put these values into the formula and solve for distance.
[tex]46=\frac{distance}{3.2}\\46*3.2=\frac{distance}{3.2}*3.2\\d=147.2[/tex]
Alex traveled 147.2 miles.
in april 2005, roland mailed a package from his local post office in Albermarle, NC ro a friend in fishers, Indiana for $2.76 per ounce. The first class domestic rate at the time was $.23 per ounce. Write and solve an equation to determine the weight of the package
The weight of the package is 12 ounces
How to solve an equation to determine the weight of the package?The given parameters are
Total amount = $2.76
Domestic rate = $.23 per ounce
The equation to determine the weight of the package is represented as:
Weight = Total amount/Domestic rate
Substitute the known values in the above equation
Weight = 2.76/.23
Evaluate the quotient
Weight = 12
Hence, the weight of the package is 12 ounces
Read more about unit rates at:
https://brainly.com/question/19493296
#SPJ1
Which element of expectancy theory could be phrased as the question, "what’s the probability that, if i do a good job, that there will be some kind of outcome in it for me?"
The expectancy theory that could be phrased as the above question is based on the element called expectancy.
In this question,
Expectancy theory proposes that an individual will behave or act in a certain way because they are motivated to select a specific behavior over others due to what they expect the result of that selected behavior will be.
To make the connection between motivation, effort and performance, expectancy theory has three variables: Expectancy, Instrumentality and Valence.
Expectancy theory consists of three basic components:
(1) The employee's expectancy that working hard will lead to his or her desired level of performance;
(2) The employee's expectancy that working hard will thus ensure that rewards will follow; and
(3) Whether or not the employee's perception that the outcome of working hard is worth the effort or value associated with hard work.
VIE expectancy theory is often formulated using the equation MF (motivational forces) = V (valence) x I (instrumentality) x E (expectancy).
From the definition, the probability that, "if i do a good job, that there will be some kind of outcome in it for me" is comes under the element expectancy(E).
Hence we can conclude that the expectancy theory that could be phrased as the above question is based on the element called expectancy.
Learn more about expectancy theory here
https://brainly.com/question/20709949
#SPJ4
How should I solve this?
The parallel sides AB, PQ, and CD, gives similar triangles, ∆ABD ~ ∆PQD and ∆CDB ~ ∆PQB, from which we have;
[tex] \frac{1}{x} + \frac{1}{y}= \frac{1}{z}[/tex]
Which method can be used to prove the given relation?From the given information, we have;
∆ABD ~ ∆PQD∆CDB ~ ∆PQBAccording to the ratio of corresponding sides of similar triangles, we have;
[tex] \frac{x}{z} = \mathbf{\frac{BD}{QD} }[/tex]
[tex] \frac{y}{z} = \frac{BD}{ BQ} [/tex]
Which gives;
[tex] \mathbf{\frac{y}{z}} = \frac{BD }{ BD - Q D} [/tex]
[tex] \frac{z}{y} = \frac{BD - QD }{ BD } = 1 - \frac{Q D }{ BD}[/tex]
QD × x = BD × z
BD × z = (1 - QD/BD) × y = y - (QD × y/BD)
Therefore;
BD × z = y - (QD × y/BD)
BQ × y = y - (QD × y/BD)
BQ × y = y - (z × y/x) = y × (1 - z/x)
(1 - z/x) = BQ
BD × z = y × (1 - z/x)
BD = (y × (1 - z/x))/z
Therefore;
QD × x = y × (1 - z/x)
(BD-BQ) × x = y × (1 - z/x)
(BD-(1 - z/x)) × x = y × (1 - z/x)
BD = (y × (1 - z/x))/x + (1 - z/x)
BQ + QD = (1 - z/x) + (y × (1 - z/x))/x
BD = BQ + QD
(y × (1 - z/x))/x + (1 - z/x) = (y × (1 - z/x))/z
(1 - z/x)×(y/x + 1) =(1 - z/x) × y/z
Dividing both sides by (1 - z/x) gives;
y/x + 1 = y/z
Dividing all through by y gives;
(y/x + 1)/y = (y/z)/y
1/x + 1/y = 1/zTherefore;
[tex] \frac{1}{x} + \frac{1}{y}= \frac{1}{z}[/tex]
Learn more about characteristics similar triangles here:
https://brainly.com/question/1799826
#SPJ1
Express the volume of the box as a
polynomial in the variable x
The volume of the box as a polynomial in the variable x is x(12 - 2x)(7 - 2x)
How to determine the volume?The complete question is added as an attachment
From the attached image, we have:
Length = 12 - 2x
Width = 7 - 2x
Height = x
The volume is calculated as:
Volume = Length * Width * Height
Substitute the known values in the above equation
Volume = (12 - 2x) * (7 - 2x) * x
This gives
Volume = x(12 - 2x)(7 - 2x)
Hence, the volume of the box as a polynomial in the variable x is x(12 - 2x)(7 - 2x)
Read more about polynomial at:
https://brainly.com/question/4142886
#SPJ1
A mathematical prodigy wishes to put 2 of his indistinguishable IMO gold medals and 2 of his indistinguishable IPhO gold medals in one row. How many distinct arrangements are possible
This relates to combination and permutations and the number of distinct arrangements that are possible will be 6.
How to illustrate the information?From the information, a mathematical prodigy wishes to put 2 of his indistinguishable IMO gold medals and 2 of his indistinguishable IPhO gold medals in one row.
Let 1 = IMO
Let 0 = IPho.
Therefore, the number of arrangement will be:
0011
0110
1100
1010
0101
1001
Therefore, the number of arrangement will be 6.
Learn more about permutations and combination on:
brainly.com/question/4658834
#SPJ1
I need help with trigonometry
Again..
Answer: C. - √2
Step-by-step explanation:
Given requirements from the question
Find the exact value of sec 135°
Convert into common trigonometric expression
sec = secant
secx = 1 / cosx
Therefore, sec 135° = 1 / (cos 135°)
Find the reference angle
135° is quite a complicated angle, but it is possible to simplify it by finding its reference angle.
Since 135° is in the second quadrant, its reference angle will be
180° - 135° = 45°
However, we need to consider the positive and negative.
In a coordinate plane, the clue is (A S T C), which corresponds to the positive values in each quadrant. In the QI, "All" are positive. In the QII, sine is positive. In the QIII, the tangent is positive. In QIV, cosine is positive.
Therefore, when the value of cosine is in QII, it is negative.
The reference angle = - (1 / cos 45°)
Determine the final value
Given that the reference angle = - (1 / cos 45°)
cos 45° = 1 / √2
Therefore:
sec 135° = - (1 / cos 45°) = - (1 / (1 / √2)) = [tex]\boxed{-\sqrt{2} }[/tex]
Hope this helps!! :)
Please let me know if you have any questions
The formula for __________________________ is x = t e or raw score (x) equals the true score (t) plus error (e).
The formula for Observed Score is; Observed score (X) = True score (T) + measurement error (e)
What is the observed score formula?The formula for Observed Score is;
Observed score (X) = True score (T) + measurement error (e)
1) True score is the score that would be obtained if an individual took a test an infinite amount of times and those test scores were averaged. The concept of true score is theoretical because you can't give someone something an infinite number of times.
2) Standard error of measurement is the standard deviation of multiple test scores (how far the sample mean of the data is likely to be from the true population mean).
The less random error (e) in the measure, the more the observed score X approximates the true score T.
Thus;
Observed score = True Score + Error
Read more about Observed Score at; https://brainly.com/question/13700221
#SPJ1
The diameter of a circle graphed in the xy-plane has endpoints at (-2, -1) and (4,7). Which of the following is an equation of the circle?
A (x + 1)² + (y + 1)² = 25
B (x + 1)² + (y + 1)² = 100
C (x - 1)² + (y - 3)² = 100
D (x - 1)² + (y - 3)² = 25
The required equation of the circle whose endpoint of the diameter (-2, -1) and (4,7) is (x - 1)² + (y - 3)² = 25 . Option D is correct.
Given that,
The endpoints of the circle's diameter (-2, -1) and (4,7).
The equation of the circle is to be determined.
The circle is the locus of a point whose distance from a fixed point is constant i.e center (h, k). The equation of the circle is given by
(x - h)² + (y - k)² = r².
where h, k is the coordinate of the circle's center on the coordinate plane and r is the circle's radius.
Center of the circle(h, k) = (-2 + 4)/2 ,(-1+7)/2
= 1 , 3
Radius of the circle,
[tex]r^2 =\sqrt{(1+2)^2+(3+1)^2}\\r^2 =\sqrt{9+16} \\r^2 = 25[/tex]
Equation of the circle with center (1, 3)
[tex]r^2 = (x - 1)^2 + (y-3)^2[/tex]
[tex]25 = (x - 1)^2 + (y-3)^2[/tex]
Thus, the required equation of the circle whose endpoint of the diameter (-2, -1) and (4,7) is (x - 1)² + (y - 3)² = 25 . Option D is correct.
Learn more about circle here:
brainly.com/question/11833983
#SPJ1
Choose the numbers that are terminating decimals. Select all that apply.
A. 0.032
B. 0.999...
C. 0.525
D. 0.75
Answer: A, C, and D
Step-by-step explanation:
A terminating decimal is a decimal that has an end. In other words, [tex]\frac{1}{4} =0.25[/tex] is one, but [tex]\frac{1}{3} =0.3333...[/tex] is not.
✓ A. 0.032
✗ B. 0.999...
✓ C. 0.525
✓ D. 0.75
The image of the point P(-3, 2) under the translation (2/1) is
The image of the point P(-3, 2) under the translation T<2,1> is (-1, 3)
How to determine the image of the translation?The coordinate point is given as:
P = (-3, 2)
The translation is given as
T<2,1>
The translation can be represented as:
(x, y) = (x + 2, y + 1)
So, we have the following equation
P' = (-3 + 2, 2 + 1)
Evaluate the sum
P' = (-1, 3)
Hence, the image of the point P(-3, 2) under the translation T<2,1> is (-1, 3)
Read more about translation at
https://brainly.com/question/21369817
#SPJ1